Skip to main content
Mathematics LibreTexts

4.3: Integrals of Functions with Branch Cuts

  • Page ID
    76221
  • \( \newcommand{\vecs}[1]{\overset { \scriptstyle \rightharpoonup} {\mathbf{#1}} } \) \( \newcommand{\vecd}[1]{\overset{-\!-\!\rightharpoonup}{\vphantom{a}\smash {#1}}} \)\(\newcommand{\id}{\mathrm{id}}\) \( \newcommand{\Span}{\mathrm{span}}\) \( \newcommand{\kernel}{\mathrm{null}\,}\) \( \newcommand{\range}{\mathrm{range}\,}\) \( \newcommand{\RealPart}{\mathrm{Re}}\) \( \newcommand{\ImaginaryPart}{\mathrm{Im}}\) \( \newcommand{\Argument}{\mathrm{Arg}}\) \( \newcommand{\norm}[1]{\| #1 \|}\) \( \newcommand{\inner}[2]{\langle #1, #2 \rangle}\) \( \newcommand{\Span}{\mathrm{span}}\) \(\newcommand{\id}{\mathrm{id}}\) \( \newcommand{\Span}{\mathrm{span}}\) \( \newcommand{\kernel}{\mathrm{null}\,}\) \( \newcommand{\range}{\mathrm{range}\,}\) \( \newcommand{\RealPart}{\mathrm{Re}}\) \( \newcommand{\ImaginaryPart}{\mathrm{Im}}\) \( \newcommand{\Argument}{\mathrm{Arg}}\) \( \newcommand{\norm}[1]{\| #1 \|}\) \( \newcommand{\inner}[2]{\langle #1, #2 \rangle}\) \( \newcommand{\Span}{\mathrm{span}}\)\(\newcommand{\AA}{\unicode[.8,0]{x212B}}\)

    When we consider multiple-valued functions, the path in a contour integral can contain a point on a branch cut of the integrand involved. The next two examples illustrate this.


    Example 1

    Let \(C\) be the semicircular path from \(z_{0}=3\) to \(z_{1}=-3\). That is \(z\left ( \theta  \right )=3e^{i\theta }\), with \(0\leq \theta \leq \pi \). Here we would like to evaluate the integral

    \(\begin{eqnarray}\label{ex-01}
                I = \int_C z^{1/2} dz.
                \end{eqnarray}\)

    To do so, we need to choose a particular branch of the multiple-valued function \(z^{1/2}\). For example, we will use the principal branch

    \(\begin{eqnarray}
                |z|> 0, \; -\pi \lt \textbf{Arg}\,(z)\lt \pi.
                \end{eqnarray}\)

    In this case, notice that although the principal branch of \(z^{1/2}=exp\left ( \frac{1}{2} Log\,z\right )\) is not defined at the end point \(z_{1}=-3\) of the contour \(C\), the integral \(I\) nevertheless exists.

    Use the following applet to explore the value of \(I\) for the given contour \(C\). Just drag the points \(z_{0}\) and \(z_{1}\) to the corresponding values. You can also select other contours and explore what happens when they cross the branch cut \(\left \{ z:x=0\,and\,y\leq 0 \right \}\).

     

    INTERACTIVE GRAPH

     

    As you already have figured it out, the integral (1) exists because the integrand is piecewise continuous on \(C\). To confirm this, observe that when \(z\left ( \theta  \right )=3e^{i\theta }\), then

    \(f\left ( z\left ( \theta  \right ) \right )=exp\left ( \frac{1}{2}ln3+i\theta  \right )=\sqrt{3}e^{i\theta /2}\).

    The left-hand limits of the real and imaginary components of the function

    \(\begin{eqnarray*}
                f\left(z(\theta)\right)z’(\theta) &=& \sqrt{3} e^{i\theta/2} \cdot 3ie^{i\theta } \\
                &=& 3 \sqrt{3} i \,e^{i3\theta/2} \\
                &=& -3\sqrt{3} \sin \frac{3\theta }{2} + i\, 3\sqrt{3} \cos \frac{3\theta }{2} \quad (0\leq \theta \lt
                \pi)\\
                \end{eqnarray*}\)

    at \(\theta =\pi \) exist. That is

    \(\begin{eqnarray*}
                \lim_{\theta \rightarrow \pi+}-3\sqrt{3} \sin \frac{3\theta }{2}= 3\sqrt{3}
                \quad \text{ and } \quad \lim_{\theta \rightarrow \pi+} 3\sqrt{3} \cos \frac{3\theta }{2} = 0.\\
                \end{eqnarray*}\)

    This means that \(f(z(θ))z′(θ)\) is continuous on the closed interval \(0 \leq \theta \leq \pi\) when its value at \(\theta = \pi\) is defined as \(3\sqrt{3}\). Therefore

    \(\begin{eqnarray*}
                I &=& \int_C f\left(z(\theta)\right)z’(\theta)\, d\theta= 3\sqrt{3} i \int_0^{\pi} e^{i\theta /2} d\theta
                = 3\sqrt{3} i\Bigg[ \bigg.\frac{2}{3i}e^{i3\theta /2}\bigg|_{0}^{\pi} \Bigg] \\
                &=& 3\sqrt{3} i\Bigg[ -\frac{2}{3i}\left( 1+i\right)\Bigg]= -2\sqrt{3}(1+i).
                \end{eqnarray*}\)

    Exercise \(\PageIndex{1}\)

    Exercise 1: Evaluate \(\int_C z^{1/2}\, dz\) for the contour \(C: z(\theta)= e^{i\theta}\), with \(-\pi\leq \theta \leq \pi\). You can use the applet to confirm your results.

    Remark 1: You can not evaluate the integral for the contour \(C: z(t)= e^{it}\), with \(0 \leq t \leq 2\pi\). Why?

    Remark 2: Notice that \(\int_C z^{1/2}\, dz=0\) for any circle not intersecting the branch cut \(\{z\,:\, x =0 \text{ and } y \leq 0\}\). Why?


    Example 2

    Consider the principal branch

    \(\begin{eqnarray}\label{br}
                f(z)=z^{i} = \exp \left[ i \,\text{Log } z \right]\; \text{ with }\; |z|>0,\, -\pi\lt \text{Arg } z \lt \pi 
                \end{eqnarray}\)

    and \(C\) the upper half circle from \(z=−1\) to \(z=1\); that is, \(z(t)=-e^{-i \pi t}\) with \(0≤t≤1\).

    Upper half circle
    Figure 1: \(z(t)=-e^{-i \pi t}\), with \(0≤t≤1\).

    It is not difficult to verify that

    \(\begin{eqnarray}\label{ex-02}
                I=\int_C z^{i}\,dz &=& \frac{1+e^{-\pi}}{2}\,(1-i).
                \end{eqnarray}\)

    Use the following applet to confirm this. You can also analize \(\int_C z^idz\) for other contours.

     

    INTERACTIVE GRAPH

     

    Remark 3: Notice that \(\int_C z^{i}\, dz=0\) for any circle not intersecting the branch cut \(\{z\,:\, x =0, y \leq 0\}\). Why?

    In general, we can calculate \(\int\) for any contour \(C\) from \(z=−1\) to \(z=1\) lying above the real axis. We just need to find an antiderivative of \(z^{i}\). Unfortunately, we can not use the principal branch, defined in (3), since this branch is not even defined at \(z=−1\). But the integrand can be replaced by the branch

    \(z^{i} = \exp \left[ i \,\text{log } z \right]\; \text{ with }\;  |z|>0,\, -\frac{\pi}{2}\lt \text{arg } z \lt \frac{3\pi}{2}.\)

    since it agrees with the integrand along \(C\).

    Smooth curve and Tangent vector
    Figure 2: Principal branch.
    Non smooth
    Figure 3: An appropiate branch.

    Using an antiderivative of this new branch, we obtain

    \(\begin{eqnarray}\label{value}
                \int_{-1}^{1} z^i dz&=& \Bigg.\frac{z^{i+1}}{i+1}\Bigg|_{-1}^{1} =  \frac{1+e^{-\pi}}{2}\,(1-i),
                \end{eqnarray}\)

    which is the same value as (4).

    Exercise \(\PageIndex{2}\)

    Evaluate \(\bigg.\frac{z^{i+1}}{i+1}\bigg|_{-1}^{1}\) to confirm the value of (5).

     


    This page titled 4.3: Integrals of Functions with Branch Cuts is shared under a CC BY-NC-SA license and was authored, remixed, and/or curated by Juan Carlos Ponce Campuzano.

    • Was this article helpful?